Đến nội dung

NHoang1608

NHoang1608

Đăng ký: 21-02-2017
Offline Đăng nhập: 24-04-2019 - 21:32
****-

#686636 Tuyển tập tạp chí THTT theo từng năm (năm 2003-2010)

Gửi bởi NHoang1608 trong 05-07-2017 - 22:05

Các file pdf đã được mình gộp các số trong năm lại và tạo bookmarks cho từng mục đề ra kì này và lời giải cho từng tháng, và một số bài viết quan trọng.
 

Năm 2010 https://www.dropbox....T 2010.pdf?dl=0
Năm 2009 https://www.dropbox....T 2009.pdf?dl=0
Năm 2008 https://www.dropbox....m 2008.pdf?dl=0
Năm 2007 https://www.dropbox....T 2007.pdf?dl=0
Năm 2006 https://www.dropbox....T 2006.pdf?dl=0
Năm 2005 https://www.dropbox....T 2005.pdf?dl=0
Năm 2004 https://www.dropbox....T 2004.pdf?dl=0
Năm 2003 https://www.dropbox....T 2003.pdf?dl=0
 
Nguồn:Tổng hơp từ các báo THTT trên trang web LTTK  :D




#686632 Tìm Min $P=\sqrt{\frac{x^3}{x^3+8y^3}...

Gửi bởi NHoang1608 trong 05-07-2017 - 21:28

$1$ cách tự nhiên ta có (theo bạn @viet9a14124869) 

$\sqrt{\frac{x^{3}}{x^{3}+8y^{3}}}=\sqrt{\frac{x^{4}}{(x^{2}+2xy)(x^{2}-2xy+4y^{2})}} \geq \sqrt{\frac{4x^{4}}{(2x^{2}+4y^{2})^{2}}}=\frac{x^{2}}{x^{2}+2y^{2}}$

$\sqrt{\frac{4y^{3}}{y^{3}+(x+y)^{3}}}=\sqrt{\frac{4y^{4}}{(2y^{2}+xy)(x^{2}+xy+y^{2})}} \geq \frac{2y^{2}}{\frac{3y^{2}+x^{2}+2xy}{2}}\geq \frac{4y^{2}}{4y^{2}+2x^{2}}=\frac{2y^{2}}{2y^{2}+x^{2}}$

Cộng vế theo vế ta có $P \geq 1$




#686567 Bất đẳng thức trong số học

Gửi bởi NHoang1608 trong 05-07-2017 - 15:29

1 cách khác nữa cho II) 2) nhưng không hay cho lắm.

 

Từ cách trên ta có $x+1=u^{2}$ và $4x+3=3v^{2}$ 

 $\Rightarrow 3\mid 4x+3 \Rightarrow 3\mid x \Rightarrow x=3k$

 Suy ra $3k+1=u^{2}, 4k+1=v^{2}$ 

 Đặt $k=8l+r$ $(0\leq r \leq 7)$ thì $24l+3r+1=u^{2}, 32l+4r+1 = v^{2}$

 Ta có $u^{2},v^{2} \equiv 0,1,4 (mod 8)$ suy ra $3r+1,4r+1 \equiv 0,1,4 (mod 8)$ suy ra $r=0$ 

 Suy ra $k=8l \Rightarrow 24l+1=u^{2} \Rightarrow l-1 \equiv 0,1,4 (mod 5) \Rightarrow l \equiv 1,2,0 (mod 5)$

Lại có $n=2x+1= 2.3k+1= 2.3.8l+1= 48l+1$ 

 Vì $l\equiv 0,1,2 (mod 5)$ nên $l=3,4$ loại, thử trực tiếp $l=0,1,2,5,6$ thì thấy vô lí vậy $l\geq 7$

Hay $n=48l+1 \geq 48.7+1= 337$




#686565 Bất đẳng thức trong số học

Gửi bởi NHoang1608 trong 05-07-2017 - 15:16

II) 2)

 Đặt $(n+1)(2n+1)=6a^{2}$ $\Rightarrow n\equiv 1 (mod 2) \Rightarrow n=2x+1$ $( x\in \mathbb{N*})$.

 Suy ra $(x+1)(4x+3)=3a^{2}$

 

Mà $(x+1;4x+3)=(4x+4;4x+3)=1$ từ đó suy ra 

$\begin{cases} x+1=u^{2} (*) \\ 4x+3=3v^{2} \end{cases}$   ($u,v \in \mathbb{N}$) Vì loại đi trường hợp $\begin{cases} x+1=3u^{2}  \\ 4x+3=v^{2} \end{cases}$ bởi $4x+3$ không thể là số chính phương.

$\Rightarrow 4u^{2}-1=3v^{2}$

$\Rightarrow (2u-1)(2u+1)=3v^{2}$

Nhận thấy tiếp là $(2u-1;2u+1)=1$ vì $2u-1;2u+1$ là $2$ số lẻ liên tiếp.

$\Rightarrow \begin{cases} 2u-1=p^{2} \\ 2u+1=3q^{2} \end{cases}$

 hoặc $\begin{cases} 2u-1=3p^{2} \\ 2u+1=q^{2} \end{cases}$

($p,q\in \mathbb{N}$)

Xét trường hợp $\begin{cases} 2u-1=3p^{2} \\ 2u+1=q^{2} \end{cases}$

 $\Rightarrow 3p^{2}+2=q^{2}$ (Vô lí)

Xét trường hợp còn lại $\begin{cases} 2u-1=p^{2} (1) \\ 2u+1=3q^{2} (2) \end{cases}$

Từ phương trình $(1)$ ta có $p$ là số lẻ. Từ đó ta đặt $p=2t+1, t\in \mathbb{N}$ 

Suy ra $2u-1=(2t+1)^{2} \Rightarrow u=2t^{2}+2t+1$  $(3)$

 

Từ $(*), (1),(3)$ thì ta có $n=2x+1=2u^{2}-1=2(2t^{2}+2t+1)^{2}-1$

Xét $t=1$ thì $n$ không thỏa mãn đề bài.

Xét $t \geq 2 \Rightarrow n\geq 2(2.4+2.2+1)^{2}-1= 337$ 

Hay ta có điều phải chứng minh




#686510 ĐỀ THI THPT CHUYÊN TRẦN PHÚ HẢI PHÒNG.

Gửi bởi NHoang1608 trong 04-07-2017 - 22:17

Cách khác cho Bài 5.1

 

Theo định lí $Fermat$ nhỏ thì ta có $x^{16} \equiv 0,1 (mod 17)$

Tương tự thì ta cũng có  $y^{16} \equiv 0,1 (mod 17)$ và $z^{16} \equiv 0,1 (mod 17)$

Mà $2017 \equiv 11 (mod17)$

Xét tất cả trường hợp số dư của $x,y,z$ khi chia cho $17$ thì ta thấy đều không thỏa mãn với đề bài.

Điều phải chứng minh




#686503 ĐỀ THI THPT CHUYÊN TRẦN PHÚ HẢI PHÒNG.

Gửi bởi NHoang1608 trong 04-07-2017 - 21:24

        SỞ GIÁO DỤC VÀ ĐÀO TẠO              TUYỂN SINH VÀO LỚP 10 THPT CHUYÊN

                   HẢI PHÒNG                                                       TRẦN PHÚ

                                                                                  

                                                                                  Thời gian làm bài:150 phút

 

Bài 1. (2,0 điểm)

a) Cho biểu thức $Q=(\frac{1}{\sqrt{1}-1}-\frac{2}{x-1}).(\frac{x+\sqrt{x}}{\sqrt{1}+1}-\frac{1-\sqrt{x}}{\sqrt{x}-x})$  (với $x>0$; $x$ khác $1$)

     1) Rút gọn biểu thức $Q$.

     2) Tìm các giá trị của $x$ để $Q=-1$.

b) Cho phương trình $x^{2}-2(m-1)x-2017m^{2}-1=0$. Tìm $m$ để phương trình có $2$ nghiệm phân biệt $x_{1}<x_{2}$ thỏa mãn $\left | x_{1} \right |-\left | x_{2} \right |=2018$

 

Bài 2. (2,0 điểm)

a) Giải phương trình $\sqrt{x+1}-\sqrt{x-7}=\sqrt{12-x}$.

b) Giải hệ phương trình $\begin{cases} x^{3}+xy^{2}-10y=0 \\ x^{2}+6y^{2}=10 \end{cases}$

 

Bài 3. (3,0 điểm)

         Cho tam giác $ABC$ nhọn nội tiếp đường tròn $(O)$. $Y$ trên cạnh $CA$. $Z$ trên cạnh $AB$ sao cho $\widehat{AZY}>90^{\circ}$. $I$ là tâm đường tròn ngoại tiếp tam giác $AYZ$, $S$ là giao điểm khác $A$ của $AI$ và đường tròn $(O)$.

a) Chứng minh $\widehat{SAC}=\widehat{AZY}-90^{\circ}$.

b) Gọi $X$ là giao điểm của $YZ$ và $BC$, $M$ là giao điểm khác $Y$ của các đường tròn ngoại tiếp tam giác $AYZ$ và $CXY$. Chứng minh rằng $M$ là điểm nằm trên đường tròn $(O)$.

b) Gọi $J,K$ là tâm các đường tròn ngoại tiếp tam giác $BZX$ và $CXY$, $T$ là giao điểm của $AI$ và $BJ$. Chứng minh $6$ điểm $T,O,M,Ị,J,K$ cùng nằm trên một đường tròn.

 

Bài 4. (1,0 điểm)

         Cho các số dương $a,b,c$ chứng minh rằng

                $\frac{a^{4}}{b^{3}(c+2a)}+\frac{b^{4}}{c^{3}(a+2b)}+\frac{c^{4}}{a^{3}(b+2c)} \geq 1$.

 

Bài 5. (2,0 điểm)

a) Chứng minh rằng không tồn tại các số tự nhiên $x,y,z$ thỏa mãn $x^{16}+y^{16}+2017=z^{16}$

b) $A$ và $B$ chơi một trò chơi, $A$ chơi trước. Ban đầu có $n$ viên sỏi. Trong mỗi lượt chơi của mình, người chơi sẽ lấy $4,5$ hoăc $7$ viên sỏi. Qúa trình đó tiếp tục như vậy. Ai đến lượt chơi của mình mà không thể lấy thêm sỏi là thua cuộc. Biết cả hai đều là chơi thông minh, chưng minh nếu $n$ có dạng $11k+l$ với $k,l \in \mathbb{N}, 0 \leq l \leq 3$ thì $B$ thắng cuộc.                                                                             




#686402 Tìm $n\in N$ để $13.3^6+3^7+3^n$ là số chính phương

Gửi bởi NHoang1608 trong 03-07-2017 - 22:21

Thấy bài này lạ mắt :)

Bằng cách thử trực tiếp ta có $n\geq7$. Lại có $3^{6}(16+3^{n-6})= a^{2}$ suy ra $16+3^{n-6}=b^{2}$.

Mà $n$ lẻ thì $b^{2}\equiv 3 (mod4)$ (Vô lí). Suy ra $n$ chẵn $\Rightarrow n-6$ chẵn.

Hay $16=(b-3^{\frac{n-6}{2}})(b+3^{\frac{n-6}{2}})$ 




#686333 $\boxed{\text{TOPIC}}$ Ôn thi học si...

Gửi bởi NHoang1608 trong 03-07-2017 - 12:45

Nghi vấn bạn @duylax2412 vào pic Ams để cóp lời giải :))

Bạn chưa có căn cứ thì không nên nói vậy. Với lại các bài bđt của bạn mình thấy là khá khó chưa phù hợp cho hs lớp 8 bây giờ mới còn bỡ ngỡ vs BĐT. 1 điều nx là mình nghĩ mọi người khối trên không nên giải bài trong topic này, để cho các bạn khối 9 tự làm. Và chỉ ra đề hoặc giải 1 bài toán đã được giải rồi theo hướng khác.

Để topic hữu ích thì mình nghĩ mọi người nên đưa ra các bài toán từ phần căn thức => pt vô tỉ =>hpt => số học => hình => bđt và cuối cùng là tổ hợp (2 phần bđt và tổ hợp là 2 phần khó cần thêm thời gian để các bạn lp 9 làm quen nên để ở cuối). 




#686267 Số nguyên dương biểu diễn dưới dạng tổng 2 số chính phương.

Gửi bởi NHoang1608 trong 02-07-2017 - 21:13

Bài toán :Chứng minh rằng nếu số nguyên dương $x$ thỏa mãn $x=a^{2}+b^{2}$ $(a;b \in \mathbb{N})$ với $(a;b)=1$ thì tồn tại $2$ số nguyên $y;z$ sao cho $z^{2}+1=xy$




#686242 Tài liệu về cấp và số mũ đúng của 1 số nguyên.

Gửi bởi NHoang1608 trong 02-07-2017 - 15:15

Mình thấy khá ít tài liệu về 2 vấn đề này hoặc mình không tìm ra được.  Vậy nên ai có tài liệu về chúng thì chia sẻ cho mọi người nhá :)




#686055 $\boxed{\text{TOPIC}}$ Ôn thi học si...

Gửi bởi NHoang1608 trong 30-06-2017 - 20:38

Cho topic thêm xôm a góp thêm vài bài số hoc :) 

 

7. Cho 2 số nguyên dương $a>1,n$ và số nguyên tố $p$ thỏa mãn $a^{n}+1 = p$. Chứng minh rằng $n$ là lũy thừa của 2.

8. Tìm số tự nhiên $n$ sao cho $A=n^{2005}+n^{2006}+n^{2}+n+2$ là 1 số nguyên tố.




#686005 Đề thi toán chuyên - chuyên KHTN ĐHQG HÀ Nội vòng 2 2017

Gửi bởi NHoang1608 trong 30-06-2017 - 12:32

phải chăng phần từ giả thiết bạn từng gặp.

Giả thiết này mình chưa từng gặp nhưng mình thấy nó quen khi 2 đại lượng $abc$ và$ab+bc+ca$ cộng với nhau liên tưởng đến HĐT $(a+1)(b+1)(c+1)=abc+ab+bc+ca+a+b+c+1$. HĐT này mình hay dùng trong số hoặc BĐT nên quen thoy :P




#685924 ĐỀ THI TUYỂN SINH VÀO LỚP 10 THPT NĂM HỌC 2017-2018 TRƯỜNG THPT CHUYÊN LÊ QUÝ...

Gửi bởi NHoang1608 trong 29-06-2017 - 14:32

Bài 5. 

  Trước tiên ta có bổ đề sau:

 Với $x,y,z > 0$ thỏa mãn $xyz \leq 1$ thì ta có $\frac{x}{y}+\frac{y}{z}+\frac{z}{x} \geq x+y+z$

Chứng minh bổ đề.

      Áp dụng bất đẳng thức $AM-GM$ ta có $\frac{x}{y}+\frac{x}{y}+\frac{y}{z} \geq 3\sqrt[3]{\frac{x^{2}}{yz}} = 3x. \sqrt[3]{\frac{1}{xyz}} \geq 3x$

       Hoàn toàn tương tự thì $\frac{y}{z}+\frac{y}{z}+\frac{z}{x} \geq 3y$ và $\frac{z}{x}+\frac{z}{x}+\frac{x}{y} \geq 3z$

Cộng 3 BĐT trên và chia 3 cho cả 2 vế thì ta có $\frac{x}{y}+\frac{y}{z}+\frac{z}{x} \geq x+y+z$.

Chứng minh bài toán.

BĐT đã cho tương đương với $ \frac{x(1-y^{3}}{y^{3}}+x +\frac{y(1-z^{3})}{z^{3}}+y+ \frac{z(1-x^{3})}{x^{3}}+z \geq x+y+z$

                                            $\Leftrightarrow \frac{x}{y^{3}}+\frac{y}{z^{3}}+\frac{z}{x^{3}} \geq x+y+z$

Áp dụng BĐT $AM-GM$ ta có $\frac{x}{y^{3}}+x+x \geq 3.\frac{x}{y}$

                         Tương tự $\frac{y}{z^{3}}+y+y \geq 3.\frac{y}{z}$ và $\frac{z}{x^{3}} + z+z \geq 3. \frac{z}{x}$

Cộng 3 BĐT trên và áp dụng bổ đề thì 

      $\frac{x}{y^{3}}+\frac{y}{z^{3}}+\frac{z}{x^{3}} + 2(x+y+z) \geq \frac{x}{y}+\frac{y}{z}+\frac{z}{x} \geq x+y+z $

 $\Leftrightarrow \frac{x}{y^{3}}+\frac{y}{z^{3}}+\frac{z}{x^{3}} \geq x+y+z $

Từ đây suy ra đpcm.




#685921 ĐỀ THI TUYỂN SINH VÀO LỚP 10 THPT NĂM HỌC 2017-2018 TRƯỜNG THPT CHUYÊN LÊ QUÝ...

Gửi bởi NHoang1608 trong 29-06-2017 - 14:15

Bài 2.2 Giả sử $b^{2}-4ac$ là số chính phương thì ta có $b^{2}-4ac = k^{2}$ với $k\in \mathbb{N}$.

 

  Ta có $4a. \overline{abc}= 4a(100a+10b+c) = 400a^{2}+40ab+4c = 400a^{2}+40ab+b^{2}- (b^{2}-4c)= (20a+b)^{2}-k^{2}= (20a+b-k)(20a+b+k)$

 Suy ra $\overline{abc} \mid (20a+b+k)(20a+b-k)$

           $\Rightarrow \overline{abc} \mid 20a+b+k$ hoặc $\overline{abc} \mid 20a+b-k$

   2 TH trên đều vô lí vì rõ ràng $\overline{abc}= 100a+10b+c > 20a+b+k > 20+b-k $

 Tóm lại điều giả sử là sai hay $b^{2}-4ac$ không phải là số chính phương.                                                                  




#685842 Turkey TST 2017

Gửi bởi NHoang1608 trong 28-06-2017 - 17:54

Bài 1

Không mất tính tổng quát giả sử $m\geq n$.

Vì $p^{3}$ chỉ có các ước nguyên dương sau $1,p,p^{2},p^{3}$ cho vậy chỉ có các trường hợp sau xảy ra.

 

TH1: $m^{3}+n=p^{3}, n^{3}+m= 1$ dễ thấy trường hợp này vô lí vì $m,n$ nguyên dương.

TH2: $m^{3}+n=p^{2}, n^{3}+m= p$

Ta có $m^{3}+n-n^{3}-m= p^{2}-p$

         $\Rightarrow (m-n)(m^{2}+mn+n^{2}-1) = p(p-1)$

         $\Rightarrow (m-n)(m^{2}+mn+n^{2}-1) \vdots p$

Xét $(m-n) \vdots p$ thì $(m-n) \vdots (n^{3}+m)$ dễ thấy mâu thuẫn vì $m,n$ nguyên dương.

Vậy $(m^{2}+mn+n^{2}-1) \vdots p$ $(1)$

Tương tự $(m^{3}+n+n^{3}+m) \vdots p$

             $\Rightarrow (m+n)(m^{2}-mn+n^{2}+1) \vdots p$

Ta xét TH $n>1$.  Nếu $(m+n) \vdots p$ thì vô lí vì $m+n < n^{3}+m= p$

Vậy $(m^{2}-mn+n^{2}+1) \vdots p$ $(2)$

Từ $(1)(2)$ thì ta có $2(m^{2}+n^{2}) \vdots p$ và $(mn-2) \vdots p$

Vì $(p;2)=1$ cho nên $(m^{2}+n^{2}) \vdots p$ $(*)$

Từ $(mn-2) \vdots p$ suy ra $(mn-2) \vdots (n^{3}+m)$

                              $\Rightarrow (mn^{3}-2n^{2}) \vdots (n^{3}+m)$

                              $\Rightarrow (mn^{3}+m^{2}- m^{2}-2n^{2}) \vdots (n^{3}+m)$

                              $\Rightarrow (m^{2}+2n^{2}) \vdots (p)= n^{3}+m$ $(**)$

Từ $(*)(**)$ thì $(m^{2}+2n^{2}-m^{2}-n^{2}) \vdots p$

                        $\Rightarrow n^{2} \vdots (p)= n^{3}+m$ (Vô lí vì $m,n$ nguyên dương)

 Bây giờ xét TH $n=1$ thì $(m+1)^{2}(m^{2}-m+1)=p^{3}$

Suy ra $m+1=p , m^{2}-m+1 =p^{2}$

           $\Rightarrow (m+1)^{2}= m^{2}-m+1$

           $\Rightarrow m=2$

Thử lại $m=2,n=1$ ta thấy đúng khi $p=3$.